An algebra problem by Vaibhav Prasad

Algebra Level 2

Find the minimum value of a 2 + b 2 + c 2 { a }^{ 2 }+{ b }^{ 2 }+{ c }^{ 2 } for positive numbers a , b , c a,b,c satisfying the constraint a + b + c = 6 a+b+c=6 .


The answer is 12.

This section requires Javascript.
You are seeing this because something didn't load right. We suggest you, (a) try refreshing the page, (b) enabling javascript if it is disabled on your browser and, finally, (c) loading the non-javascript version of this page . We're sorry about the hassle.

33 solutions

Curtis Clement
Feb 20, 2015

a 2 + b 2 + c 2 = ( a + b + c ) 2 2 ( a b + b c + a c ) = 36 2 ( a b + b c + a c ) ( 1 ) a^2 +b^2+c^2 = (a+b+c)^2 - 2(ab+bc+ac) = 36 - 2(ab+bc+ac) \ (1) Now we must maximise 2(ab +bc +ac) in order to minimise our expression. So using the Cauchy Schwarz inequality : ( a b + a c + b c ) 2 ( a 2 + b 2 + c 2 ) ( a 2 + b 2 + c 2 ) (ab+ac+bc)^2 \leq\ (a^2+b^2+c^2)(a^2+b^2+c^2) m a x ( a b + a c + b c ) = a 2 + b 2 + c 2 \Rightarrow\ max(ab+ac+bc) = a^2 +b^2+c^2 Now we can substitute this into (1) to give: 36 = 3 ( a 2 + b 2 + c 2 ) m i n ( a 2 + b 2 + c 2 ) = 12 36 = 3(a^2 + b^2 + c^2) \therefore\ min(a^2+b^2+c^2) = \boxed{12} w h e n a = b = c = 2 \large when \ a=b=c= 2

( a + b + c ) 2 ( a 2 + b 2 + c 2 ) ( 1 2 + 1 2 + 1 2 (a+b+c)^{2} \leq (a^{2}+b^{2}+c^{2})(1^{2}+1^{2}+1^{2} gives the answer directly.

Joel Tan - 6 years, 3 months ago

Log in to reply

Yep, that's even more elegant and swift approach.

Kishlaya Jaiswal - 6 years, 3 months ago

Ah yes. Direct application of CS to the sequences { a , b , c } \{a,b,c\} and { 1 , 1 , 1 } \{1,1,1\} saves time and makes this problem a one-liner. Good thinking.

Prasun Biswas - 6 years, 3 months ago

We can also see that

a 2 + b 2 + c 2 a b b c c a = 1 2 ( ( a b ) 2 + ( b c ) 2 + ( c a ) 2 ) 0 a^2+b^2+c^2-ab-bc-ca = \frac{1}{2}\left((a-b)^2+(b-c)^2+(c-a)^2\right) \geq 0

Thus,

a 2 + b 2 + c 2 a b b c c a = 3 2 ( a 2 + b 2 + c 2 ) 1 2 ( a + b + c ) 2 0 a^2+b^2+c^2-ab-bc-ca = \frac{3}{2}(a^2+b^2+c^2)-\frac{1}{2}(a+b+c)^2 \geq 0

a 2 + b 2 + c 2 ( a + b + c ) 2 3 = 12 \Rightarrow a^2+b^2+c^2 \geq \frac{(a+b+c)^2}{3} = 12

Thus. ( a 2 + b 2 + c 2 ) m i n = 12 \left( a^2+b^2+c^2 \right)_{min} = 12 which occurs when a = b = c = 2 a=b=c=2

Kishlaya Jaiswal - 6 years, 3 months ago

Log in to reply

The identity stated at the beginning follows from Lagrange's Identity , I presume?

Prasun Biswas - 6 years, 3 months ago

Log in to reply

It's simple factorization

a 2 + b 2 + c 2 a b b c c a = 1 2 ( a 2 + b 2 2 a b + b 2 + c 2 2 b c + c 2 + a 2 2 a c ) = 1 2 ( ( a b ) 2 + ( b c ) 2 + ( c a ) 2 ) 0 a^2+b^2+c^2-ab-bc-ca = \frac{1}{2}\left(a^2+b^2-2ab+b^2+c^2-2bc+c^2+a^2-2ac\right) \\ =\frac{1}{2}\left((a-b)^2+(b-c)^2+(c-a)^2\right) \geq 0

Kishlaya Jaiswal - 6 years, 3 months ago

Log in to reply

@Kishlaya Jaiswal Yes, I know that. I'm saying that the idea isn't much trivial. I guessed that you recalled the identity using the n = 3 n=3 case of the Lagrange's Identity. :)

Prasun Biswas - 6 years, 3 months ago

Log in to reply

@Prasun Biswas Unfortunately, no!

Actually, I didn't know about that identity earlier.

Thanks to you for telling me about it.

Kishlaya Jaiswal - 6 years, 3 months ago

Log in to reply

@Kishlaya Jaiswal Ah, mention not. :)

Prasun Biswas - 6 years, 3 months ago

Log in to reply

@Prasun Biswas It just struck my mind that a 2 + b 2 + c 2 a b + b c + c a a^2+b^2+c^2 \geq ab+bc+ca is none other than Re-arrangement inequality. That makes it pretty much more trivial. ¨ \ddot \smile

Kishlaya Jaiswal - 6 years, 3 months ago

Log in to reply

@Kishlaya Jaiswal Ah yes! I forgot about that.

Prasun Biswas - 6 years, 3 months ago

The problem becomes trivial if one uses Jensen's Inequality on the convex function f ( x ) = x 2 f(x)=x^2 with x = a , b , c > 0 x=a,b,c\gt 0 .

Prasun Biswas - 6 years, 3 months ago

Log in to reply

ah yes of course! f ( x 1 ) + f ( x 2 ) + . . . + f ( x n ) n f ( x 1 + x 2 + . . . + x n n ) \frac{f(x_1)+f(x_2) +...+f(x_n)}{n}\geq\ f (\frac{x_1 +x_2 +...+x_n}{n}) a 2 + b 2 + c 2 3 ( a + b + c ) 2 9 \Rightarrow\frac{a^2+b^2+c^2}{3}\geq\frac{(a+b+c)^2}{9} Very nice idea :)

Curtis Clement - 6 years, 3 months ago

That's just quadratic mean - arithmetic mean .

Calvin Lin Staff - 6 years, 3 months ago

i don't know the Cauchy Schwarz inequality, so won't it be much more easier to find the possibilities and then work them out?

Ansh Bhatt - 6 years, 3 months ago

Log in to reply

It would take long time.

Harsh Shrivastava - 6 years, 3 months ago

a, b, c should not be equal to the same thing, so they'll must call the variables just a. This doesn't make sense, they don't put a, b and c to be the same number...

Charles Grenier - 5 years, 6 months ago

Guys I see this in a much more simpler light. All one has to do is equally distribute the sum amongst each of the numbers to get the least value for the sum of their squares. This should also work for 4, 5, 6 or n numbers. Try it. In fact this should work for the sum of the cubes or the 4th powers and so on. For example what is the minimum value of a^2 + b^2 + c^2 + d^2 when a + b + c + d = 8. It is true when they are equally distributed and a = b = c = d = 2. Intuitive solution is better and easier, eh?!! ;-).

Swadeep Pillarisetti - 5 years, 1 month ago

Log in to reply

It is not always true that the max/min must occur when "equally distribute the sum amongst each of the numbers". See inequalities with strange equality conditions for examples.

In this case, the "intuitive" solution happens to work, but you have to explain what makes it work.

Calvin Lin Staff - 5 years ago

If a=1, b=1, c=4 then a+b+c=6 & sum of squares of a, b, c is 16 which is highest number 1+1+4=6.......... (1 1)+(1 1)+(4*4)=16

Aamir Malik - 6 years, 3 months ago

Log in to reply

We have to find the minimum value. @Aamir Malik

Harsh Shrivastava - 6 years, 3 months ago

wtf i thought a,b,c have to be distinct

William G. - 5 years, 10 months ago

Log in to reply

I assume you came upon the realization that they did not have to be distinct when you tried 14 and it was wrong. Once it was obvious that they didn't, I immediately jumped to "We gotta get rid of this 3!!!"

Dan Daughtry - 5 years, 5 months ago
Fredrik Meyer
Feb 22, 2015

Every solution here is very algebraic. There is a geometric way to see this. The conditions imply that (a,b,c) must lie somewhere on the triangle spanned by (6,0,0),(0,6,0) and (0,0,6). The expression a 2 + b 2 + c 2 a^2+b^2+c^2 is the square of the norm of the vector (a,b,c). So we are looking for the vector of least length. By "picture" or by symmetry considerations, we see that this occurs on the midpoint of the triangle, or in other words, when a = b = c a=b=c , which can only happen when a = b = c = 2 a=b=c=2 . Hence the answer is 4 3 = 12 4 \cdot 3 = 12 .

Hi Fredrik, Your approach looks interesting. Could you please show some more light on how do you relate the triangle and vector in this sum ?

Kanagaraj n.n - 5 years, 10 months ago

Hey Frederick. Where can I learn more about your method? Perhaps a link? Thank you (:

Stef Mlad - 5 years, 5 months ago
Harsh Shrivastava
Feb 19, 2015

By Cauchy Schwarz inequality ,

a 2 + b 2 + c 2 ( a + b + c ) 2 1 + 1 + 1 a^2 + b^2 + c^2 \geq \dfrac{(a+b+c)^2}{1+1+1}

\implies a 2 + b 2 + c 2 12 a^2 + b^2 + c^2 \geq 12 .

This form of Cauchy-Schwarz Inequality is also known as Titu's Lemma (also called "Cauchy-Schwarz in Engel Form").

Prasun Biswas - 6 years, 3 months ago

C O R R E C T ! ! ! CORRECT !!!

Vaibhav Prasad - 6 years, 3 months ago

Log in to reply

Try this

Harsh Shrivastava - 6 years, 3 months ago

By the way did you try my VALENTINE problem ????????????????

https://brilliant.org/problems/stupidest-valentine-problem-ever/?group=xUAvDrMJo83o&ref_id=625760

Vaibhav Prasad - 6 years, 3 months ago

Log in to reply

Nope, can you give me its link?

Harsh Shrivastava - 6 years, 3 months ago

Log in to reply

link di to hai : ) :)

Vaibhav Prasad - 6 years, 3 months ago

Log in to reply

@Vaibhav Prasad Oh yeah!

Good problem,trying to solve!!

Harsh Shrivastava - 6 years, 3 months ago

Nicely done - I used the same inequality but a slightly longer method

Curtis Clement - 6 years, 3 months ago
Phạm Nhung
Feb 25, 2015

Good approach...

Rahul Singh - 6 years, 3 months ago
M M
Feb 22, 2015

a 2 + b 2 + c 2 = ( a + b + c ) 2 2 ( a b + a c + b c ) a^2+b^2+c^2=(a+b+c)^2-2(ab+ac+bc) and a b + a c + b c a 2 + b 2 + c 2 ab+ac+bc \leq a^2+b^2+c^2 (by AM-GM inequality,Cauchy or simple rearrengment) , we have

a 2 + b 2 + c 2 ( a + b + c ) 2 2 ( a 2 + b 2 + c 2 ) a^2+b^2+c^2 \geq (a+b+c)^2-2(a^2+b^2+c^2)

3 ( a 2 + b 2 + c 2 ) ( a + b + c ) 2 = 36 3(a^2+b^2+c^2) \geq (a+b+c)^2 = 36

a 2 + b 2 + c 2 12 a^2+b^2+c^2 \geq 12

Equality occurs when a = b = c = 2 a=b=c = 2

Good Solution....... : ) :)

Vaibhav Prasad - 6 years, 3 months ago

That's the way I solved it...nice!

Bhupendra Jangir - 6 years, 3 months ago

I used LaGrange :v

Its quite simple:

restriction : a+b+c=6

derivate in a,b and c both sides and lambda.

λ:2a

λ:2b

λ:2c

So a=b=c, aplying in the restriction

a+a+a=6

3a=6

a=2=b=c

(2)^2+(2)^2+(2)^2=4+4+4=12

Azadali Jivani
Feb 21, 2015

1 + 2 + 3 = 6 & 1^2 + 2^2 + 3^2 = 14
1 + 1 + 4 = 6 & 1^2 +1^2 + 4^2 = 18
2 + 2 + 2 = 6 & 2^2 + 2^2 + 2^2 = 12 Ans.
Note: as per given condition, values of a, b, c should be minimum and here less than 2 is not possible.


There is a interesting geometric solution:

  1. a+b+c=6 is a plane. The equation is 1. a + 1. b + 1. c 6 = 0 1.a + 1.b + 1.c - 6 = 0
  2. a, b, c are axes.
  3. Thus, a 2 + b 2 + c 2 a^2 + b^2 + c^2 is is the square of the norm of the vector that goes from (0,0,0) to the plane.
  4. A minimal a 2 + b 2 + c 2 a^2 + b^2 + c^2 is square of the norm of the perpendicular vector that goes from (0,0,0) to the plane.
  5. Thus, a minimal a 2 + b 2 + c 2 a^2 + b^2 + c^2 is the square of distance of plane 1. a + 1. b + 1. c 6 = 0 1.a + 1.b + 1.c - 6 = 0 to (0,0,0).
  6. The formula for a distance d is: d = 1.0 + 1.0 + 1.0 6 1 2 + 1 2 + 1 2 d = \frac{ |1.0 + 1.0 + 1.0 - 6|}{ \sqrt{1^2 + 1^2 + 1^2}}
  7. Thus, d = 6 3 d = \frac{6}{\sqrt{3}} and d 2 = a 2 + b 2 + c 2 = 12 d^2 = a^2 + b^2 + c^2 = 12
Ayran Michelin
Jul 26, 2015

Isn't it soooo obvious that the Minimum value comes with the lesser square possible? In other words:

If you need to satisfy a + b + c = 6 with numbers wich squares are the lesser, it obviously will be the middle number as possible (in this case 2 for the three, cause in the others ways you can take 1 or 0 in one incognite, but have to use numbers much biggers in one of the others to compensate: example 0, 1, 5; 0, 0, 6; and so on - even 1, 2, 3 the square of 3 is much bigger than the square of 2!! i hope i could express my thinking). Isn't it logical?

Why all those counts?

Leonard Kho
Feb 21, 2015

Anyone else who used Lagrange Multiplier? :)

Can you show your work for Lagrange Multiplier?

Calvin Lin Staff - 6 years, 3 months ago

Log in to reply

Seems to me that mr. Kho has a point. Using superior mathematics for such a problem is a little discordant! I agree to this opinion!

Andi Popescu - 6 years, 3 months ago

Using the extended version of the AM-GM Inequality, the QM-AM-GM-HM Inequality, we know that (using the first two inequalities)

i = 1 n ( x i ) 2 n i = 1 n x i n \displaystyle \sqrt{\dfrac{\sum\limits_{i=1}^{n} {(x_i)}^{2} }{n}} \geq \dfrac{\sum\limits_{i=1}^{n} x_i}{n}

For n = 3 n = 3 , this simplifies to

a 2 + b 2 + c 2 3 a + b + c 3 \displaystyle \sqrt{\dfrac{a^2 + b^2 + c^2}{3}} \geq \dfrac{a+b+c}{3}

Since a + b + c = 6 a+b+c=6 ,

a 2 + b 2 + c 2 3 6 3 = 2 \displaystyle \sqrt{\dfrac{a^2 + b^2 + c^2}{3}} \geq \dfrac{6}{3} = 2

a 2 + b 2 + c 2 3 2 2 \displaystyle \implies \dfrac{a^2 + b^2 + c^2}{3} \geq 2^2

a 2 + b 2 + c 2 12 \displaystyle \implies a^2 + b^2 + c^2 \geq 12

Therefore, the minimum value of a 2 + b 2 + c 2 = 12 a^2 + b^2 + c^2 = \boxed{12} with equality occuring when a = b = c = 2 a=b=c=2 .

Anito Anto
Oct 2, 2016

When I think a + b + c = 6, the minimum value of a,b and c is 2 that is a=b=c=2, therefore substituting the values, 4+4+4=12

Raakin Kabir
Aug 29, 2016

All you need to really do is see what is the smallest integers that can satisfy the second equation. That would be... a = 2 b = 2 c = 2 2 + 2 + 2 = 6 2^2 + 2^2 + 2^2 = 4 + 4 + 4 = 8 + 4 = 12

Justin Bonanno
Jun 25, 2016

I used Lagrange multipliers to solve the problem, went rather smoothly.

Guys I see this in a much more simpler light. All one has to do is equally distribute the sum amongst each of the numbers to get the least value for the sum of their squares. This should also work for 4, 5, 6 or n numbers. Try it. In fact this should work for the sum of the cubes or the 4th powers and so on. For example what is the minimum value of a^2 + b^2 + c^2 + d^2 when a + b + c + d = 8. It is true when they are equally distributed and a = b = c = d = 2. Intuitive solution is better and easier, eh?!! ;-).

Ta3iapxHs .
Dec 16, 2015

(a^2+b^2+c^2)'=2a+2b+2c=2(a+b+c)

Max value for a+b+c=6 so,

2*6=12

(I don't see this solution so I must be wrong somewhere..)

Jason Simmons
Nov 6, 2015

Using the method of Lagrange Multipliers, this problem can be easily approached. First let's define the function to be minimized as f ( a , b , c ) = a 2 + b 2 + c 2 f(a,b,c)=a^{2}+b^{2}+c^{2} and the constrain function to be g ( a , b , c ) = a + b + c = 6 g(a,b,c)=a+b+c=6 By Lagrange Multipliers the gradient of the minimized function is proportional to the constrain function, thus f ( a , b , c ) = λ g ( a , b , c ) \nabla f(a,b,c)=\lambda \nabla g(a,b,c) the gradient of the minimized function is f ( a , b , c ) = 2 < a , b , c > \nabla f(a,b,c)= 2<a,b,c> and the gradient of the constrained function is g ( a , b , c ) = λ < 1 , 1 , 1 > \nabla g(a,b,c)= \lambda<1,1,1>

We are now left with three equations - 2 x = λ 2x=\lambda - 2 y = λ 2y=\lambda - 2 z = λ 2z=\lambda which implies that x = y = z x=y=z . Because of this we can now substitute y and z into our constraint equation yielding the equation

x + x + x = 6 x = 2 x+x+x=6 \Rightarrow x=2

Now that we finally have revealed the values of x, y, and z we can substitute it into our function to be minimized leaving us with

2 2 + 2 2 + 2 2 = 12 2^{2}+2^{2}+2^{2}=\boxed{12}

Felipe Perestrelo
Aug 22, 2015

Without knowing much of hardcore math, such as the Cauchy Schwarz inequality, I took a more intuitive and empirical approach.

Let's assume m i n ( a 2 + b 2 + c 2 ) min(a^{2} + b^{2} + c^{2}) happens for a = b = c = 2 a = b = c = 2 . Now, if that's wrong, I should be able to obtain a lower value for a 2 + b 2 + c 2 a^{2} + b^{2} + c^{2} by locking one of the variables, let's say a a , and raising another one, let's say b b , which would naturally drop c s c's value equivalently, since we have a + b + c = 6 a + b + c = 6 .

But we know that the exponential function of power of 2 will grow higher than it will diminish for a giving e p s i l o n epsilon if we have a positive base.

Therefore any raise in b and equivalent drop in c c would cause a 2 + b 2 + c 2 a^{2} + b^{2} + c^{2} to raise its value.

Paul Saligumba
Jul 11, 2015

Can't relate to the discussion 😅😅😅

Freddie Kalaitzis
Apr 29, 2015

The minimum value for a 2 + b 2 + c 2 a^2 + b^2 + c^2 is also the minimum value for a 2 + b 2 + c 2 \sqrt{a^2 + b^2 + c^2} . The solution is where the sphere of radius a 2 + b 2 + c 2 \sqrt{a^2 + b^2 + c^2} touches the plane a + b + c = 6 a + b + c =6 . Since a,b,c contribute equally to the linear equation then they must be equal to 2, which gives a 2 + b 2 + c 2 = 12 a^2 + b^2 + c^2 = 12 .

Aditya Dhabu
Feb 27, 2015

You will get the min value when all the terms (a, b, c) are minimum. And that happens when a=b=c =6

Joel Tan
Feb 27, 2015

By Cauchy-Schwarz,

36 = ( a + b + c ) 2 ( a 2 + b 2 + c 2 ) ( 1 2 + 1 2 + 1 2 ) = 3 ( a 2 + b 2 + c 2 ) 36=(a+b+c)^{2} \leq (a^{2}+b^{2}+c^{2})(1^{2}+1^{2}+1^{2})=3 (a^{2} +b^{2}+c^{2})

Thus the answer is at least 12. But a = b = c = 2 a=b=c=2 gives the value of 12. Thus 12 is the answer.

A=B=C=2 GIVES THE MINIMUNM VALUE THE CONCEPT COMES FROM AM>=GM

Andi Popescu
Feb 24, 2015

It is known that : a 2 + b 2 + c 2 a b + b c + c a ( 1 ) a^2 + b^2 + c^2 \geq\ ab + bc + ca \ (1) The equal sign is for a = b = c. This comes from: ( a b ) 2 0 (a - b)^2 \geq 0 and 2 similars to, for b with c and c with a. Equal, evidently obtained, for a = b, b = c and respectively c = a. Summing the 3 inequalities & simplifying by 2 we obtain the (1) relation . We have: ( a + b + c ) 2 = 6 2 a 2 + b 2 + c 2 + 2 ( a b + b c + c a ) = 36 ( 2 ) (a+b+c)^2 = 6^2\rightarrow \ a^2+b^2+c^2+2(ab+bc+ca) = 36 \ (2) Substituting (1) in (2) we obtain: a 2 + b 2 + c 2 + 2 ( a 2 + b 2 + c 2 ) 36 a^2+b^2+c^2+2(a^2+b^2+c^2)\geq 36 & simplifying by 3 we obtain the answer: a 2 + b 2 + c 2 12 \rightarrow \ a^2+b^2+c^2 \geq 12 Equal is obtained for: a = b = c

All the three numbers are greater than zero,and their sum is 6.So,the three nunumbers can be 1,2and 3 or 2,2 and 2.In the second case,we get the least number i.e 12! and voila I got the answer with no algebra.....: P

Why greater than zero? And why integers? Evidently you've got something, but no the answer! ;)

Andi Popescu - 6 years, 3 months ago
Gamal Sultan
Feb 23, 2015

Let u =(1, 1, 1), v = (a, b, c) be two vectors, the acute angle between them be x

The scalar product of u, v = u . v = a + b + c = 6

Norm of u = ||u|| = square root of 3

Norm of v = ||v|| = square root of (a^2 + b^2 + c^2)

cos x = ( u . v )/||u||.||v|| = 6/square root of 3(a^2 + b^2 + c^2)

cos x is less than or equal to 1

cos x is more than or equal to 0 ...........(x is acute)

Then

a^2 + b^2 + c^2 is more than or equal 12

Then

The minimum value of a^2 + b^2 + c^2 is 12

Lalima Rao
Feb 23, 2015

a+b+c=6
6/3 =2 2 2+2 2+2*2=4+4+4 =12 a,b,c are three numbers so if divided by total value get the variables value.

Mohammed Hssein
Feb 21, 2015

Using the jensen's inequality for the function X--> X^2 we conclude immediately

This is very not hard. We can also use other inequalities like Holder's inequality or the T2 lemma....

Mohammed Hssein - 6 years, 3 months ago
Md Moniruzzaman
Feb 21, 2015

2 + 2 + 2 = 6 & 2^2 + 2^2 + 2^2 = 12 Ans

Ashwin Upadhyay
Feb 21, 2015

can you plz explain the title @Vaibhav Prasad

I used the title because this problem is one of the basic applications of the Cauchy-Schwarz inequality, as shown in the solution of a friend of mine, Harsh Shrivastava

Vaibhav Prasad - 6 years, 3 months ago

Log in to reply

o.k :p nice title

Ashwin Upadhyay - 6 years, 3 months ago
Kana Cummings
Feb 20, 2015

Guess and check 1+2+3=6 (1^{2})+(2^{2})+(3{^2})=14 2+2+2+6 3*(2{^2})=12 minimum value=12

Brock Brown
Feb 20, 2015

Why not use Python ?

1
2
3
4
5
6
7
8
from itertools import product
tests = xrange(1, 5)
smallest = 1000000
for a,b,c in product(tests,repeat=3):
    if a+b+c == 6:
        if a*a+b*b+c*c < smallest:
            smallest = a*a+b*b+c*c
print "Answer:", smallest

This is programmation, is this Microsoft Visual Studio???

Charles Grenier - 5 years, 6 months ago
Ayush Garg
Feb 20, 2015

Since the AM of nth powers is greater than nth power of AM of the terms... Therefore {a^2 + b^2 +c^2}/3 > 4 and hence the result..

We can exploit the result HM >= AM Using the result we can get. : a²+b²+c²>= 12

A Former Brilliant Member - 3 years, 9 months ago

0 pending reports

×

Problem Loading...

Note Loading...

Set Loading...